31
$\begingroup$

Starting with a single stick of unit length, a point $p \in (0, 1)$ is picked uniformly at random along the stick and the stick is snapped, producing two sticks of length $p$ and $1-p$.

At each next stage, a stick is picked uniformly at random, and a point is picked uniformly at random along the length of that stick, and it is snapped.

Question: After n snaps, what is the expected length of the longest remaining stick?

Remarks:

Myself and a friend of mine did some simulations and found some pretty unexpected results. The expected value after $500$ splits is approximately $0.2098$, which is massive for that many splits, at least intuitively.

On the other hand, it can be proven rather easily that the expected value does go to $0$ as $n \to \infty$. But the decay seems extremely slow.

$\endgroup$
15
  • 1
    $\begingroup$ Sort of similar process: mathoverflow.net/q/415252 $\endgroup$ Sep 13, 2022 at 13:01
  • 3
    $\begingroup$ In case you haven't seen the stick-breaking constructions of the beta and Dirichlet processes, you may find the analysis interesting. $\endgroup$ Sep 13, 2022 at 13:18
  • 2
    $\begingroup$ There is a natural interpolation between this question and the one Sam Hopkins links: Let $0 \leq p \leq \infty$. If the stick has been broken into $n$ pieces, of lengths $x_1$, $x_2$, ..., $x_n$, then pick up the $j$-th piece with probability $x_j^p/\sum_i x_i^p$, and break that piece at a uniformly random position. This question is $p=0$, and the other is $p = \infty$. Unfortunately, only $p=1$ seems easy to analyze. $\endgroup$ Sep 13, 2022 at 13:25
  • 2
    $\begingroup$ Combining those, I think we have $E(X_{n+1}) \geq (1-\tfrac{1}{4n}) E(X_n)$ and hence $E(X_{n+1}) \geq (1-\tfrac{1}{4n})(1-\tfrac{1}{4n-4})(1-\tfrac{1}{4n-8}) \cdots \approx \tfrac{1}{n^{1/4}}$. $\endgroup$ Sep 13, 2022 at 13:41
  • 2
    $\begingroup$ @SamHopkins Right, that's why I thought it would be tractable. But then I tried to work out what I thought the maximum gap between n id uniform variables would be, and it still seems hard. $\endgroup$ Sep 13, 2022 at 14:03

4 Answers 4

18
$\begingroup$

It seems that the length of the longest stick is of order $n^{2\sqrt{2}-3} = n^{-0.171\ldots}$ as $n\to\infty$. This follows from a discrete-time analogue of the homogeneous fragmentation process, see chapter 1.5 of J. Bertoin, Random fragmentation and coagulation processes. Vol. 102. Cambridge University Press, 2006.

Let us denote by $X_{n,0} \geq X_{n,1} \geq \cdots \geq X_{n,n}$ the ordered sizes of the sticks after $n$ snaps. The first result we need is the following.

Lemma: $\chi_{n}(p) := \mathbb{E}\left[\sum_{i=0}^n X_{n,i}^p\right] = \frac{1}{n!}\left(\frac{2}{1+p}\right)_n,$ where $(a)_n=a(a+1)\cdots (a+n-1)$ is the rising Pochhammer symbol.

Proof: We have \begin{align*} \mathbb{E}\left[\sum_{i=0}^n X_{n,i}^p\middle| X_{n-1,0},\ldots\right] &= \frac{1}{n}\sum_{k=0}^{n-1}\Big(\mathbb{E}\left[x^p + (X_{n-1,k}-x)^p\middle|X_{n-1,k}\right]+\sum_{\substack{i=0\\i\neq k}}^{n-1}X_{n-1,i}^p\Big)\\ &= \frac{1}{n}\left(\frac{2}{p+1}+n-1\right)\sum_{i=0}^{n-1}X_{n-1,i}^p, \end{align*} where $x$ is uniform in $(0,X_{n-1,k})$. Hence $\chi_n(p) = \frac{1}{n}\left(\frac{2}{p+1}+n-1\right) \chi_{n-1}(p)$. Together with $\chi_0(p)=1$, this gives the claimed formula for $\chi_n(p)$. $\square$

Following Corollary 1.4 in Bertoin's book, we note that \begin{equation} n^{\frac{p-1}{p+1}} \chi_n(p) = \frac{1}{\Gamma\left(\frac{2}{p+1}\right)} + O(n^{-1}). \end{equation} In particular it is bounded for any $p>-1$. Since $X_{n,0}^p < \sum_{i=0}^n X_{n,i}^p$, we deduce that $n^{\frac{p-1}{p+1}}X_{n,0}^p$ is bounded as $n\to\infty$. Hence \begin{equation} \limsup_{n\to\infty} \frac{\log X_{n,0}}{\log n} \leq -\frac{1}{p}\frac{p-1}{p+1} \leq -\frac{1}{\bar{p}}\frac{\bar{p}-1}{\bar{p}+1} = 2\sqrt{2}-3, \end{equation} because the maximum $\bar{p}$ of $-\frac{1}{p}\frac{p-1}{p+1}$ is achieved at $\bar{p} = 1+\sqrt{2}$. Similarly one can derive a matching lower bound by noting that \begin{equation} X_{n,0}^\epsilon \geq \frac{\sum_{i=0}^{n}X_{n,i}^p}{\sum_{i=0}^{n}X_{n,i}^{p-\epsilon}} \end{equation} for any $\epsilon>0$, which implies \begin{equation} \liminf_{n\to\infty} \frac{\log X_{n,0}}{\log n} \geq - \frac{\frac{p-1}{p+1} - \frac{p-\epsilon-1}{p-\epsilon+1}}{\epsilon} \end{equation} for any $\epsilon>0$ and $0<p<\bar{p}$. Letting $\epsilon$ approach $0$ and $p$ approach $\bar{p}$, we thus have \begin{equation} \liminf_{n\to\infty} \frac{\log X_{n,0}}{\log n} \geq -\frac{2}{(1-\bar{p})^2}=2\sqrt{2}-3. \end{equation} We may therefore conclude that \begin{equation} \lim_{n\to\infty} \frac{\log X_{n,0}}{\log n} =2\sqrt{2}-3. \end{equation}

$\endgroup$
5
  • $\begingroup$ I think this is as close to a complete answer as one can hope for - asymptotics in $n$. So I have accepted the answer, though others are still very welcome to contribute. Thank you for the solution! $\endgroup$
    – Nate River
    Sep 15, 2022 at 15:48
  • $\begingroup$ I am confused about the lower bound. You have $X_{n,0}^{\epsilon} \geq \frac{\sum_k X_{n,k}^p}{\sum_k X_{n,k}^{p-\epsilon}}$. How do you go from there to the lower bound on $\frac{\log X_{n,0}}{\log n}$. At first I thought you were saying that $\mathbb{B}(X_{n,0}^{\epsilon}) \geq \frac{\mathbb{E}(\sum_k X_{n,k}^p)}{\mathbb{E}(\sum_k X_{n,k}^{p-\epsilon})}$, but I don't see how to take the division outside the expectation. Also, you say the next line is only good for $0 < p < \overline{p}$, which suggests you are doing something else; could you please spell out what it is? $\endgroup$ Sep 15, 2022 at 16:31
  • $\begingroup$ Some of this is probably my discomfort with probabilistic language. When you say $\lim \text{sup} \frac{\log X_n}{\log n}$ is less than something, or $\lim \text{inf} \frac{\log X_n}{\log n}$ is more than something, you mean with probability $1$, right? As compared to $X_{n,0}^{\epsilon} \geq \frac{\sum_k X_{n,k}^p}{\sum_k X_{n,k}^{p-\epsilon}}$, which is literally true everywhere in the probability space? $\endgroup$ Sep 15, 2022 at 16:33
  • $\begingroup$ Yes, I have been skipping corners especially on the lower bound. When I find some time I will add some details. Essentially I am just translating the proof of Corollary 1.4 in the book to the discrete setting, so in the meantime if you wish (and can access the book) you could have a look there for the proper arguments. $\endgroup$ Sep 15, 2022 at 19:25
  • $\begingroup$ Thanks! At this point, I think I have found a way to fix the argument that I am satisfied with, so don't feel a need to rush on my account. Maybe I'll add it as a comment if I get some time. $\endgroup$ Sep 16, 2022 at 0:55
6
$\begingroup$

UPDATE#2. Just to illustrate how the complexity of exact expressions grows, these are the first three: \begin{split} n=1:~~~& \frac{3}{4} = 0.75\\ n=2:~~~& \frac38 + \log\frac43 = 0.662682072451781\ldots\\ n=3:~~~& \frac{5+4\pi^{2}}{24}-\log(2)^{2}+\frac{89 \log(2)}{18}-\frac{17 \log(3)}{6}+\frac{2 \log(3) \log(2)}{3}-\frac{2 \Re\,\mathrm{Li}_{2}(\tfrac{3}{2})}{3} = 0.612043787903219\ldots \end{split}

UPDATE#1. As pointed out in the comments, the recurrence for $L(n)$ derived below gives only a lower bound.


Let $L(n)$ be the expected length of the longest stick after $n$ snaps. Consider the two sticks resulted from the first snap, call them left and right. Noticing that the probability of exactly $k\in\{0,1,\dots,n-1\}$ out of the following $n-1$ snaps happening in the (descendants of) left stick equals $\frac1n$, we get a recurrence formula starting at $L(0)=1$:

\begin{split} L(n) &= \frac1n \int_0^1 {\rm d}p\sum_{k=0}^{n-1} \max\{\ pL(k),\ (1-p)L(n-1-k)\ \}\\ &= \frac1n \sum_{k=0}^{n-1} L(k) - \frac{L(k)L(n-1-k)}{2(L(k)+L(n-1-k))}. \end{split} (simplified per David E Speyer's suggestion)

Here is a sample Sage code computing $L(n)$ for $n=1..10$.

$\endgroup$
10
  • 4
    $\begingroup$ I get that $\int_{p=0}^1 \max(pA, (1-p)B) dp = \tfrac{A^2+AB+B^2}{2A+2B}$, so you can interchange the sum and the integral and remove the integral. $\endgroup$ Sep 13, 2022 at 14:02
  • 2
    $\begingroup$ @MaxAlekseyev Actually, I'm not sure why the recurrence should involve a $\text{max}$ over the expected values, seems like its the pointwise (in $\omega$) values that are relevant no? Could you clarify a bit as to how the recurrence is obtained? $\endgroup$
    – Nate River
    Sep 13, 2022 at 14:32
  • 2
    $\begingroup$ By $\omega$ I mean the random parameter, that is, the element of the probability space $\Omega$. The maximum length comes from either the left sticks descendants or the right sticks, but it seems what we want is something of the form $\mathbb E[\text{max}(\text{left}, \text{ right})]$. It is not clear to me how to move the expectation inside the max. $\endgroup$
    – Nate River
    Sep 13, 2022 at 14:40
  • 3
    $\begingroup$ I think Nate River is right. Concerning the reasoning in your comment, I cannot attach a meaning to the expectation over a set. In fact, $E\max(X,Y)>\max(EX,EY)$ if $P(X>Y)>0$ and $P(X<Y)>0$. Also, your $L(500)=0.18\ldots$ is a bit less than the simulated value $0.20\ldots$, which seems to be due to the strict inequalities of the form $E\max(X,Y)>\max(EX,EY)$. $\endgroup$ Sep 13, 2022 at 15:05
  • 2
    $\begingroup$ Hm let us be more precise - using $l$, $r$ to denote left and right, we have $\mathbb E[\text{max}(l, r)] = \mathbb E[\mathbf 1_{\Omega_1} l] + \mathbb E[\mathbf 1_{\Omega_2} r] \neq \max(\mathbb E[l], \mathbb E[r])$ in general. Noting that neither of us have used anything special about $l$ or $r$, one can just pick arbitrarily these random variables and see that this doesn’t hold in general. $\endgroup$
    – Nate River
    Sep 13, 2022 at 15:07
3
$\begingroup$

Here's a few empirical results that might be helpful, particularly in lending credence towards some of the theories suggested in other posts. I ran 10,000 simulations, and for each one, broke the stick 10,000 times, tracking the longest portion at each step. Here's the results: enter image description here

and the same data plotted on a log scale: enter image description here

You may notice a highlighted $\pm 1 \sigma$ region mentioned on the legend; the corresponding confidence interval is so tight that it's hard to see.

I added two dotted lines corresponding to two rates of decay mentioned in other posts: $d(n) \propto n^{2\sqrt{2}-3}$ and $d(n) \propto (1-\frac{1}{4n-4})d(n-1)$.

The two hypotheses are intended as asymptotic descriptions, so if we plot them directly, they have large and distracting offsets. I "fixed" that by multiplying by the appropriate constant to make the plots intersect the empirical mean at $x=1000$. (For instance, the dotted green line is actually $0.591 n^{2\sqrt{2}-3}$.)

Incidentally, we can compute our own version of the expected length after the 500th split. I ran 1 million simulations and observe an empirical mean of $0.2076537 \pm 0.0001231$. (The $\pm$ part is one standard deviation for the estimate.)

$\endgroup$
4
  • 2
    $\begingroup$ I don't understand why $n^{2 \sqrt{2} - 3} \approx n^{-0.17}$ is drawn below $\prod_{k=1}^n (1-1/(4k)) \approx n^{-0.25}$. We should have $n^{-0.25} < n^{-0.17}$. $\endgroup$ Sep 15, 2022 at 18:23
  • $\begingroup$ Good question, @DavidESpeyer ! I'll double-check... $\endgroup$ Sep 15, 2022 at 21:56
  • $\begingroup$ Ah... this seems to be an artifact of my additive "fix" to shift the plots onto the same scale. A multiplicative fit paints a different picture (with the expected ordering of the estimates). I'll run a higher-quality version and repost when it's done... $\endgroup$ Sep 15, 2022 at 23:55
  • $\begingroup$ OK, I switched from additive to multiplicative fit, and the lower bound has actually become a lower bound. I wonder if there's a more thoughtful way to go about fitting these curves. $\endgroup$ Sep 16, 2022 at 0:20
3
$\begingroup$

This is intermediate between an answer, and a long comment on Max Alekseyev's post.

As discussed in the comments below his answer, there is a flaw in his post, so that it gives a lower bound. But there's a straightforward way to progress past the flaw. Unfortunately, there's a further obstruction that I do not see how to proceed beyond.

Begin with a slight generalization. Start with a single node of length $s$, and build a random tree as follows:

  • Select a leaf node $n$ uniformly at random. Call its weight $w$.
  • Select $p\in[0,1]$ uniformly at random.
  • Attach two children to $n$ of weight $pw$ and $(1-p)w$.

Your problem asks: if $s=1$, what is the largest weight of a leaf, when there are $n$ nodes? Call this random variable $X^0_n$; in the general case, that length scales to $sX^0_n$.

As Max Alekseyev noticed, there is a natural recurrence structure to this tree. For notational convenience, suppose $n+1$ nodes. The root node has two children, weighted $P$ and $1-P$; let there be $K$ and $n-K$ nodes beneath each (inclusive); then $K$ is chosen uniformly from $\{0,1,\dots,n\}$. Take two iid copies of $X^0$, called $X^1$ and $X^2$. Since leaf nodes must descend from the children of the root, $$(X_{n+1}^0|K,P)=\max(PX^1_K,(1-P)X^2_{n-K})$$

To proceed exactly, introduce a CDF-like function. Let $F_n(t)=\mathbb{P}\left[{X_n\leq\frac{1}{t}}\right]$; then \begin{align*} (n+1)F_{n+1}(t)&=(n+1)\mathbb{P}\left[{X_n^0\leq\frac{1}{t}}\right] \\ &=\sum_{k=0}^n{\int_0^1{\mathbb{P}\left[{\max(PX^1_K,(1-P)X^2_{n-K})\leq\frac{1}{t}}\middle|{K=k,|P-p|\leq dp}\right]}} \\ &=\sum_{k=0}^n{\int_0^1{\mathbb{P}\left[{pX^1_k\leq\frac{1}{t}}\wedge{(1-p)X^2_{n-k}\leq\frac{1}{t}}\right]\,dp}} \\ &=\sum_{k=0}^n{\int_0^1{F_k(pt)F_{n-k}((1-p)t)\,dp}} \end{align*}

The finite sum is a discrete convolution, and can be eliminated by passing to generating functions. Let $\mathcal{F}(t,z)=\sum_{n=0}^{\infty}{F_n(t)z^n}$. Then $$\mathcal{F}(pt,z)\mathcal{F}((1-p)t,z)=\sum_{n=0}^{\infty}{\sum_{k=0}^n{F_k(pt)F_{n-k}((1-p)t)z^n}}$$ Integrating in $p$ recovers the recurrence from above, which simplifies to \begin{gather*} \frac{\mathcal{F}(t,z)}{\partial z}=\int_0^1{\mathcal{F}(pt,z)\mathcal{F}((1-p)t,z)\,dp} \\ \mathcal{F}(t,0)=F_0(t)=\begin{cases}t&0<t\leq1\\0&\text{otherwise}\end{cases} \end{gather*} Note that $F_n(t)=0$ for $t<0$, so that we can extend the bounds of integration to $\mathbb{R}$ without effect.

The other integral can almost be eliminated via the Fourier transform. Now let $\mathcal{G}(u,z)=\int_\mathbb{R}{\mathcal{F}(t,z)e^{uti}\,dt}$. Then \begin{align*} \frac{\partial\mathcal{G}(u,z)}{\partial z}&=\int_\mathbb{R}{\frac{\mathcal{F}(t,z)}{\partial z}e^{uti}\,dt} \\ &=\iint_{\mathbb{R}^2}{e^{upti}\mathcal{F}(pt,z)\cdot e^{u(1-p)ti}\mathcal{F}((1-p)t,z)\,d^2(p,t)} \\ &=\iint_{\mathbb{R}^2}{e^{\alpha ui}\mathcal{F}(\alpha,z)\cdot e^{\beta ui}\mathcal{F}(\beta,z)\,\frac{d^2(\alpha,\beta)}{\alpha+\beta}} \end{align*} If the $\alpha+\beta$ in the denominator could be removed (say, by tweaking the definition of $F_n$ and using an analogous integral transform), then the last line would simplify to $\mathcal{G}(u,z)^2$.

Unfortunately, I do not know an integral transform that avoids the extra $\alpha+\beta$ term. Nevertheless, let me indulge and sketch the the argument after such a gap were filled, although I'm sure the structure of the argument will come as little surprise to most readers.

Suppose the necessary integral transform has $L^2$ adjoint given by kernel $\hat{I}$ and define $$\alpha(u)=\int_1^\infty{\frac{\hat{I}(t)}{t^2}\,dt}$$ (Yes, one probably needs a little care to show that this integral converges. But it's ultimately just calculating an explicit integral.)

In the OP, you ask for \begin{align*} \mathbb{E}[X_n]&=\int_0^1{\mathbb{P}[X_n\leq t]\,dt} \\ &=\int_0^1{F_n\left(\frac{1}{t}\right)\,dt} \\ &=\int_1^\infty{F_n(u)\,\frac{du}{u^2}} \end{align*} which has generating function $$\mathcal{E}(z)=\int_1^\infty{\mathcal{F}(t,z)\,\frac{dt}{t^2}}=\int_\mathbb{R}{\overline{\alpha(u)}\mathcal{G}(u,z)\,du}$$ Thus it suffices to have an explicit formula for $\mathcal{G}$.

From the known value of $\mathcal{F}(t,0)$, $$\mathcal{G}(u,0)=\frac{1-e^{ui}(1-ui)}{u^2}$$ The differential equation above is then an ODE that uniquely determines $\mathcal{G}$ as some locally-$C^{\infty}$ function $$\mathcal{G}(u,z)=\sum_{n=0}^\infty{g_n(u)z^n}$$ Comparing coefficients of $z^n$, $$\mathbb{E}[X_n]=\int_\mathbb{R}{\overline{\alpha(u)}g_n(u)\,du}$$ and so the problem reduces to estimating an explicit integral.

$\endgroup$
4
  • 1
    $\begingroup$ Even if the $n^{-1/4}$ bound is wrong, we certainly have a lower bound of $n^{-1}$, since that is the average length of a piece! $\endgroup$ Sep 15, 2022 at 3:02
  • $\begingroup$ @DavidESpeyer: Yes; I should think the error is in my calculation, not yours. (I've screwed up problems like this before.) But I know of analogous problems for which the structure of the argument is sound; there should just be some fortuitous cancellations that end up with an integral that is $\approx n^{-1/4}$. $\endgroup$ Sep 15, 2022 at 3:05
  • $\begingroup$ @MaxAlekseyev: Ah, crap. That is the error, and I don't see a way to fix it. Thanks for wading through my equations. $\endgroup$ Sep 19, 2022 at 4:19
  • $\begingroup$ @JacobManaker: Wouldn't $F_n(t)=\sqrt t\,\mathbb{P}\left[{X_n\leq\frac{1}{\sqrt t}}\right]$ do the job by any chance? $\endgroup$ Sep 20, 2022 at 17:01

Your Answer

By clicking “Post Your Answer”, you agree to our terms of service and acknowledge you have read our privacy policy.

Not the answer you're looking for? Browse other questions tagged or ask your own question.